Problem on a set which is a subset of a finite set

  • Thread starter Thread starter issacnewton
  • Start date Start date
  • Tags Tags
    Finite Set
Click For Summary
The discussion focuses on proving that a set A, which is a subset of a finite set, is also finite. The proof attempts to establish a bijection between A and a finite set Im, showing that the number of elements in A is less than or equal to n, where n is the number of elements in the larger set In. It concludes that if A is not equal to In, then A must have fewer elements than In, reinforcing the finiteness of A. However, there are concerns about the validity of the proof, particularly regarding assumptions made in the argument. The discussion highlights the importance of definitions and theorems in validating such proofs.
issacnewton
Messages
1,035
Reaction score
37
Homework Statement
Prove that if ##n \in \mathbb{N}## and ##A \subseteq I_n##, then ##A## is finite and ##|A|\leq n ##. Furthermore, if ##A \ne I_n##, then ##|A| < n##. We are given that
$$ I_n = \big \{ i \in \mathbb{Z}^+ | i \leq n \big \} $$
Relevant Equations
Definition of a finite set and definition of bijections
Here is my attempt. Since we have to prove that ##A## is finite, we need to prove that there exists some ##m \in \mathbb{N}##, such that there is a bijection from ##A## to ##I_m##. And hence we have ##A \thicksim I_m##. Now, since there are ##n## elements in ##I_n##, number of elements in ##A## are less than or equal to ##n##. So, we have ## |A|\leq n ##. Let, ##|A| = m##. Now, we can construct a function from ##A## to ##I_n##, such that ##m## elements in ##A## are paired with first ##m## members in ##I_n##. And, we see that the first ##m## members in ##I_n## is the set ##I_m##. So, the function would be one to one and onto and hence we have ##A \thicksim I_m##. Which proves that ##A## is a finite set. Furthermore, if ##A \ne I_n##, then ##A## is a strict subset of ##I_n## and ##A## would have less number of elements than ##I_n##. So, we have ##|A| < n##. Is the proof valid ?

Thanks
 
Physics news on Phys.org
I don't know if you're allowed to use this but I think inclusion/exclusion would do it since it applies to finite sets.
 
WWGD, thanks, but is the proof presented here valid ?
 
This is a tricky issue because it depends on all the definitions, results and theorems available to you. Still, from what I have read, it seems you're assuming what you want to prove: ( bold is mine).

"Here is my attempt. Since we have to prove that A is finite, we need to prove that there exists some m∈N, such that there is a bijection from A to Im. And hence we have A∼Im. Now, since there are n elements in In, number of elements in A are less than or equal to n"
 
  • Like
Likes FactChecker
Question: A clock's minute hand has length 4 and its hour hand has length 3. What is the distance between the tips at the moment when it is increasing most rapidly?(Putnam Exam Question) Answer: Making assumption that both the hands moves at constant angular velocities, the answer is ## \sqrt{7} .## But don't you think this assumption is somewhat doubtful and wrong?

Similar threads

  • · Replies 2 ·
Replies
2
Views
1K
Replies
1
Views
2K
  • · Replies 15 ·
Replies
15
Views
3K
  • · Replies 4 ·
Replies
4
Views
1K
  • · Replies 1 ·
Replies
1
Views
2K
Replies
7
Views
2K
Replies
4
Views
1K
  • · Replies 11 ·
Replies
11
Views
2K
Replies
34
Views
3K
  • · Replies 15 ·
Replies
15
Views
2K